What is the speed of the particle at x=3.8 m?

In summary, the problem involves finding the work done by a force acting on a particle with a mass of 1.6kg, where the force is related to the position of the particle by F=0.5x^{3}. The work is given by the integral W = \int F*dx, where the limits of integration are x=3.8m and x=2.0m. The force at these points is calculated to be 27.436N and 4.000N respectively. To find the work, the definite integral of F with respect to x must be taken. The second part of the problem involves finding the speed of the particle at x=3.8m, given that at x=2
  • #1
astr0
17
0
A force acts on a particle of 1.6kg mass, the force is related to the position of the particle by F=0.5[tex]x^{3}[/tex]

Find the work done by the force as the particle moves from x=3.8 m to x=2.0 m.

By plugging each value of x into the force equation I get that:
F at 3.8 m = 27.436 N
F at 2.0 m = 4.000 N

I don't know where to go from here.
I have tried using the average of those two forces, and that was not correct.
 
Physics news on Phys.org
  • #2
How do you define work done in terms of an integral?
 
  • #3
Work = [tex]\int F*dx[/tex]
Which gives [tex]\frac{1}{2}F^{2}[/tex]
But how does that help me?
 
  • #4
astr0 said:
Work = [tex]\int F*dx[/tex]
Which gives [tex]\frac{1}{2}F^{2}[/tex]
But how does that help me?

So you know that

[tex]W= \int F dx.[/tex]

You know F in terms of some function x and they told you the limits for the x values.
 
Last edited:
  • #5
Simplified, W = integral(F dx) with the limits of integration. Therefore your F = 0.5(x^3) represents the function being integrated as with the change in x which is given for you. Thus, when you find the definite integral with the bounds given as with the function F, you have your work.
 
  • #6
I understand now. I wasn't making the connection. Thanks.
 
  • #7
astr0 said:
Work = [tex]\int F*dx[/tex]
Which gives [tex]\frac{1}{2}F^{2}[/tex]
But how does that help me?

Er that integration is done incorrectly. Integrating F with respect to dx will not give you .5F^2. You are incorrectly applying the power rule for integration...
 
  • #8
I realize that now.
 
  • #9
The second part of the problem states At x= 2.0 m the force points opposite the direction of the particle's velocity (speed is 12 m/s). What is its speed at 3.8 m?

If the force points opposite, shouldn't the particle be slowing down?
 

Related to What is the speed of the particle at x=3.8 m?

What is work done by a force?

Work done by a force is a measure of the energy transferred to an object when a force is applied and the object moves in the direction of the force.

How is work done by a force calculated?

The work done by a force is calculated by multiplying the force applied to an object by the distance the object moves in the direction of the force.

What is the unit of measurement for work done by a force?

The unit of measurement for work done by a force is joules (J). This can also be represented by Newton-meters (N·m) since force is measured in Newtons (N) and distance in meters (m).

Can work done by a force be negative?

Yes, work done by a force can be negative if the force is applied in the opposite direction to the motion of the object. This means that the object loses energy rather than gaining it.

How is the direction of work done by a force determined?

The direction of work done by a force is determined by the direction of the force and the direction of the object's motion. If the force and motion are in the same direction, the work done is positive. If the force and motion are in opposite directions, the work done is negative.

Similar threads

  • Introductory Physics Homework Help
Replies
4
Views
1K
  • Introductory Physics Homework Help
Replies
12
Views
819
  • Introductory Physics Homework Help
Replies
16
Views
578
  • Introductory Physics Homework Help
Replies
2
Views
244
  • Introductory Physics Homework Help
Replies
11
Views
1K
  • Introductory Physics Homework Help
Replies
4
Views
1K
  • Introductory Physics Homework Help
Replies
8
Views
951
Replies
1
Views
642
Replies
29
Views
1K
  • Introductory Physics Homework Help
Replies
5
Views
629
Back
Top